LSAT and Law School Admissions Forum

Get expert LSAT preparation and law school admissions advice from PowerScore Test Preparation.

 Administrator
PowerScore Staff
  • PowerScore Staff
  • Posts: 8916
  • Joined: Feb 02, 2011
|
#85114
Complete Question Explanation

Assumption. The correct answer choice is (B).

Answer choice (A):

Answer choice (B): This is the correct answer choice.

Answer choice (C):

Answer choice (D):

Answer choice (E):

This explanation is still in progress. Please post any questions below!
User avatar
 FiskPrime
  • Posts: 1
  • Joined: Apr 12, 2023
|
#100836
Assumption questions are my worst!

Why is this (B) and not (A)?

Did I make a mistake in processing an assumption question?
 Luke Haqq
PowerScore Staff
  • PowerScore Staff
  • Posts: 747
  • Joined: Apr 26, 2012
|
#100840
Hi FiskPrime!

If assumption questions are your worst, make sure to drill yourself on the Assumption Negation technique. It's not something that you want to use on all 5 answer choices, but it can be a powerful tool if you've winnowed it down to a couple options or a single option that you want to confirm. Pre-phrasing is also generally a useful tool which can help you anticipate the correct answer.

In this stimulus, the conclusion is that the mentioned concern about side effects is unfounded. The author argues that this is because the side effects can be addressed with "careful medical supervision." But what if not everyone could access careful medical supervision? The author seems to suggest that anyone can access it.

This is why (B) is correct, which we can confirm using the Assumption Negation technique. The technique involves negating the answer choice and then plugging that negated statement back into the stimulus; if it makes the stimulus fall apart/weakens it, then you know you've chosen an assumption that is required by the argument. Answer choice (B) states, "No migraine sufferers with heart disease will take the new medication except under careful medical supervision." A negation of this would be: "Some/at least one migraine sufferer will not take the new medication under medical supervision." If that were true, it would call into question the conclusion that concerns about side effects are unfounded. This confirms that it's the correct answer choice.

By contrast, the negation of answer choice (A) would be: "[It is not the case that the] new medication actually is effective when taken by patients with heart disease." If the medication were not actually effective, this wouldn't challenge the conclusion that concerns about side effects are unfounded. It might suggest it's useless to take the medication for migraines, but it doesn't address the side effects and whether or not concerns about them are unfounded.

Get the most out of your LSAT Prep Plus subscription.

Analyze and track your performance with our Testing and Analytics Package.